LSAT Explanation PT 33, S1, Q25: The number of airplanes equipped with

LSAT Question Stem

Which one of the following, if true, most seriously weakens the argument? 

Logical Reasoning Question Type

This is a Weaken question. 

Correct Answer

The correct answer to this question is D. 

LSAT Question Complete Explanation

The question type for this problem is Weaken, which means we need to find an answer choice that undermines the conclusion or the relationship between the premises and the conclusion, without attacking any premises.

First, let's analyze the argument in the passage. The author concludes that the new anticollision device is responsible for the sudden disappearance of key information from air traffic controllers' screens. The premises supporting this conclusion are that the number of airplanes equipped with the new device has increased steadily during the past two years, and during the same period, it has become increasingly common for key information to disappear suddenly from controllers' screens.

To help illustrate the argument, imagine a town where the number of people wearing a particular brand of shoes has increased over the past year. At the same time, there has been an increase in the number of foot injuries. The argument is similar to concluding that the shoes are causing the foot injuries because both events are happening simultaneously.

Our "Evaluate" question for this argument would be: "Is there any other factor that could explain the sudden disappearance of key information from controllers' screens?"

Now let's discuss each answer choice:

a) The new anticollision device has already prevented a considerable number of mid-air collisions.

- This answer choice provides another positive effect of the new device but does not weaken the argument about the disappearance of key information. It's like saying the shoes in our example are very fashionable, which doesn't address the foot injury issue.

b) It was not until the new anticollision device was introduced that key information first began disappearing suddenly from controllers' screens.

- This answer choice actually strengthens the argument by showing a correlation between the introduction of the device and the disappearance of key information. It does not weaken the argument.

c) The new anticollision device is scheduled to be moved to a different frequency within the next two to three months.

- This answer choice talks about a future event and does not address the current issue of key information disappearing. It does not weaken the argument.

d) Key information began disappearing from controllers' screens three months before the new anticollision device was first tested.

- This is the correct answer choice. It weakens the argument by showing that the effect (disappearance of key information) existed before the supposed cause (the new anticollision device) was introduced. It suggests that there might be another factor causing the disappearance of key information.

e) The sudden disappearance of key information from controllers' screens has occurred only at relatively large airports.

- This answer choice does not impact the argument, as it does not address whether the new anticollision device is causing the disappearance of key information. It merely talks about where the issue is happening, which is not relevant to the argument's validity.

Previous
Previous

LSAT Explanation PT 34, S2, Q13: Essayist: One of the claims of

Next
Next

LSAT Explanation PT 33, S1, Q4: Juan: Unlike the ancient Olympic games